Peds Exam 3- GI/ GU

Lakukan tugas rumah & ujian kamu dengan baik sekarang menggunakan Quizwiz!

379. Which question should the nurse ask the parents of a child suspected of having glomerulonephritis? 1. "Did your child fall off a bike onto the handlebars?" 2. "Has the child had persistent nausea and vomiting?" 3. "Has the child been itching or had a rash anytime in the last week?" 4. "Has the child had a sore throat or a throat infection in the last few weeks?"

79. Answer: 4 Rationale: Glomerulonephritis refers to a group of kidney disorders characterized by inflammatory injury in the glomerulus. Group A β-hemolytic streptococcal 1102 infection is a cause of glomerulonephritis. Often, a child becomes ill with streptococcal infection of the upper respiratory tract and then develops symptoms of acute poststreptococcal glomerulonephritis after an interval of 1 to 2 weeks. The assessment data in options 1, 2, and 3 are unrelated to a diagnosis of glomerulonephritis.

Which laboratory tests should the nurse prepare to draw when admitting a pediatric client with possible obstructive uropathy? Select all that apply. 1. Platelet count 2. Blood urea nitrogen (BUN) 3. Partial thromboplastin time (PTT) 4. Blood culture 5. Creatinine

Answer: 2, 5 Explanation: 1. Platelet count is drawn when a bleeding disorder is suspected. 4 Copyright © 2017 Pearson Education, Inc. 2. BUN is a serum laboratory test for kidney function. Obstructive uropathy is a structural or functional abnormality of the urinary system that interferes with urine flow and results in urine backflow into the kidneys; therefore, the BUN will be elevated. 3. PTT is drawn when a bleeding disorder is suspected. 4. A blood culture is done when an infectious process is suspected. 5. Creatinine is a serum laboratory test for kidney function. Obstructive uropathy is a structural or functional abnormality of the urinary system that interferes with urine flow and results in urine backflow into the kidneys; therefore, the creatinine will be elevated.

9 A 3-month-old infant has gastroesophageal reflux (GER) but is thriving without other complications. The mother wants to know what she can do differently to decrease the reflux. Which intervention should the nurse suggest to minimize reflux? 1. Discontinue breast-feeding immediately. 2. Increase frequency of feedings and keep them small. 3. Place the baby in prone position with the head flat. 4. Place the infant in a car seat after feeding.

Answer: 2Infants with GER should be given small, frequent feedings. After a feeding the infant should be placed in a prone position with the head of the bed elevated. A harness can be used to help maintain this position. Infant seats should be avoided because of the increased intrabdominal pressure this position creates.

The nurse is preparing to ambulate a school-age client who had an appendectomy. In addition to pharmacological pain management, the nurse can use which nonpharmacological pain-management strategy for this client? 1. A heating pad 2. A warm, moist pack 3. A pillow on the abdomen 4. An ice pack

Answer: 3 Explanation: 1. A pillow placed on the abdomen can be a nonpharmacological strategy to decrease discomfort after an appendectomy. Heat and ice are not used on the incisional area as they can impair the healing process of the wound.

10 A 10-year-old boy has been admitted with a diagnosis of "rule out appendicitis." While the nurse is conducting a routine assessment, the boy states, "It doesn't hurt anymore." The nurse suspects that: 1. The boy is afraid of going to surgery. 2. The boy is having difficulty expressing his pain adequately. 3. The appendix has ruptured. 4. This is a method the boy uses to receive attention.

Answer: 3Signs and symptoms of a ruptured appendix include fever, sudden relief from abdominal pain, guarding, abdominal distention, rapid shallow breathing, pallor, chills, and irritability.

10 A child returning to the unit after an intravenous pyelogram (IVP) has an order to drink extra fluids. When the mother asks the purpose of these fluids, the nurse responds that increased fluid intake will: 1. Overhydrate the child. 2. Increase serum creatinine levels. 3. Make up for fluid losses from NPO status before tests. 4. Flush any remaining dye from the urinary tract.

Answer: 4The additional fluids will increase urinary output, causing greater urine volume and more frequent voiding, thus flushing the dye from the urinary system. The other options do not describe the correct rationale for this intervention.

1 An 18-month-old client is brought in for a well-child visit. The parent reports feeling a lump to the right of the "bellybutton" during bathing. Initial assessments should include: (Select all that apply.) 1. Measuring weight and height. 2. Palpation of the area. 3. Routine urine testing. 4. Vital signs. 5. Questioning the parents about abuse.

1 Answers: 1, 3, 4This is the usual presentation of Wilm's tumor (nephroblastoma), and palpating the area may cause the tumor to spread. Since Wilm's tumor is a cancer of the kidney, it important to assess growth and development; kidney function; and blood pressure, which may be elevated due to increased renin production. There is no evidence of abuse or rationale for performing a neuro check.

3 A client is to begin radiation therapy after the removal of Wilms' tumor. The parent statement that indicates to the nurse a lack of understanding of related skin care would be "We will: 1. Use loose-fitting clothes on our child." 2. Protect our child from sun exposure." 3. Keep the area moist with petrolatum jelly." 4. Prevent our child from scratching the site."

3 Answer: 3Self-care during external radiation therapy includes loose-fitting clothes, gentle washing with mild soap, avoiding sun exposure, and avoiding scratching and other irritation. Any lubricant must be watersoluble, not oil-based such as petrolatum jelly.

332. The clinic nurse reviews the record of an infant and notes that the primary health care provider (PHCP) has documented a diagnosis of suspected Hirschsprung's disease. The nurse reviews the assessment findings documented in the record, knowing that which sign most likely led the mother to seek health care for the infant? 1. Diarrhea 2. Projectile vomiting 3. Regurgitation of feedings 4. Foul-smelling ribbon-like stools

332. Answer: 4 Rationale: Hirschsprung's disease is a congenital anomaly also known as congenital aganglionosis or aganglionic megacolon. It occurs as the result of an absence 1000 of ganglion cells in the rectum and other areas of the affected intestine. Chronic constipation beginning in the first month of life and resulting in pellet-like or ribbonlike stools that are foul-smelling is a clinical manifestation of this disorder. Delayed passage or absence of meconium stool in the neonatal period is also a sign. Bowel obstruction, especially in the neonatal period; abdominal pain and distention; and failure to thrive are also clinical manifestations. Options 1, 2, and 3 are not associated specifically with this disorder.

332. The clinic nurse reviews the record of an infant and notes that the primary health care provider (PHCP) has documented a diagnosis of suspected Hirschsprung's disease. The nurse reviews the assessment findings documented in the record, knowing that which sign most likely led the mother to seek health care for the infant? 1. Diarrhea 2. Projectile vomiting 3. Regurgitation of feedings 4. Foul-smelling ribbon-like stools

333. Answer: 3 Rationale: A cleft lip is a congenital anomaly that occurs as a result of failure of soft tissue or bony structure to fuse during embryonic development. After cleft lip repair, the nurse avoids positioning an infant on the side of the repair or in the prone position, because these positions can cause rubbing of the surgical site on the mattress. The nurse positions the infant on the side lateral to the repair or on the back upright and positions the infant to prevent airway obstruction by secretions, blood, or the tongue. From the options provided, placing the infant on the left side immediately after surgery is best to prevent the risk of aspiration if the infant vomits.

334. The nurse reviews the record of a newborn infant and notes that a diagnosis of esophageal atresia with tracheoesophageal fistula is suspected. The nurse expects to note which most likely sign of this condition documented in the record? 1. Incessant crying 2. Coughing at nighttime 3. Choking with feedings 4. Severe projectile vomiting

334. Answer: 3 Rationale: In esophageal atresia and tracheoesophageal fistula, the esophagus terminates before it reaches the stomach, ending in a blind pouch, and a fistula is 1001 present that forms an unnatural connection with the trachea. Any child who exhibits the "3 Cs"—coughing and choking with feedings and unexplained cyanosis—should be suspected to have tracheoesophageal fistula. Options 1, 2, and 4 are not specifically associated with tracheoesophageal fistula.

335. The nurse provides feeding instructions to a parent of an infant diagnosed with gastroesophageal reflux disease. Which instruction should the nurse give to the parent to assist in reducing the episodes of emesis? 1. Provide less frequent, larger feedings. 2. Burp the infant less frequently during feedings. 3. Thin the feedings by adding water to the formula. 4. Thicken the feedings by adding rice cereal to the formula.

335. Answer: 4 Rationale: Gastroesophageal reflux is backflow of gastric contents into the esophagus as a result of relaxation or incompetence of the lower esophageal or cardiac sphincter. Small, more frequent feedings with frequent burping often are prescribed in the treatment of gastroesophageal reflux. Feedings thickened with rice cereal may reduce episodes of emesis. If thickened formula is used, cross-cutting of the nipple may be required.

336. A child is hospitalized because of persistent vomiting. The nurse should monitor the child closely for which problem? 1. Diarrhea 2. Metabolic acidosis 3. Metabolic alkalosis 4. Hyperactive bowel sounds

336. Answer: 3 Rationale: Vomiting causes the loss of hydrochloric acid and subsequent metabolic alkalosis. Metabolic acidosis would occur in a child experiencing diarrhea because of the loss of bicarbonate. Diarrhea might or might not accompany vomiting. Hyperactive bowel sounds are not associated with vomiting.

337. The nurse is caring for a newborn with a suspected diagnosis of imperforate anus. The nurse monitors the infant, knowing that which is a clinical manifestation associated with this disorder? 1. Bile-stained fecal emesis 999 2. The passage of currant jelly-like stools 3. Failure to pass meconium stool in the first 24 hours after birth 4. Sausage-shaped mass palpated in the upper right abdominal quadrant

337. Answer: 3 Rationale: Imperforate anus is the incomplete development or absence of the anus in its normal position in the perineum. During the newborn assessment, this defect should be identified easily on sight. However, a rectal thermometer or tube may be necessary to determine patency if meconium is not passed in the first 24 hours after birth. Other assessment findings include absence or stenosis of the anal rectal canal, presence of an anal membrane, and an external fistula to the perineum. Options 1, 2, and 4 are findings noted in intussusception.

338. The nurse admits a child to the hospital with a diagnosis of pyloric stenosis. On assessment, which data would the nurse expect to obtain when asking the parent about the child's symptoms? 1. Watery diarrhea 2. Projectile vomiting 3. Increased urine output 4. Vomiting large amounts of bile

338. Answer: 2 Rationale: In pyloric stenosis, hypertrophy of the circular muscles of the pylorus causes narrowing of the pyloric canal between the stomach and the duodenum. Clinical manifestations of pyloric stenosis include projectile vomiting, irritability, hunger and crying, constipation, and signs of dehydration, including a decrease in urine output.

339. The nurse provides home care instructions to the parents of a child with celiac disease. The nurse should teach the parents to include which food item in the child's diet? 1. Rice 2. Oatmeal 3. Rye toast 4. Wheat bread

339. Answer: 1 1003 Rationale: Celiac disease also is known as gluten enteropathy or celiac sprue and refers to intolerance to gluten, the protein component of wheat, barley, rye, and oats. The important factor to remember is that all wheat, rye, barley, and oats should be eliminated from the diet and replaced with corn, rice, or millet. Vitamin supplements—especially the fat-soluble vitamins, iron, and folic acid—may be needed to correct deficiencies. Dietary restrictions are likely to be lifelong.

340. The nurse is preparing to care for a child with a diagnosis of intussusception. The nurse reviews the child's record and expects to note which sign of this disorder documented? 1. Watery diarrhea 2. Ribbon-like stools 3. Profuse projectile vomiting 4. Bright red blood and mucus in the stools

340. Answer: 4 Rationale: Intussusception is a telescoping of 1 portion of the bowel into another. The condition results in an obstruction to the passage of intestinal contents. A child with intussusception typically has severe abdominal pain that is crampy and intermittent, causing the child to draw in the knees to the chest. Vomiting may be present, but is not projectile. Bright red blood and mucus are passed through the rectum and commonly are described as currant jelly-like stools. Watery diarrhea and ribbon-like stools are not manifestations of this disorder.

341. Which interventions should the nurse include when creating a care plan for a child with hepatitis? Select all that apply. 1. Providing a low-fat, well-balanced diet. 2. Teaching the child effective hand-washing techniques. 3. Scheduling playtime in the playroom with other children. 4. Notifying the primary health care provider (PHCP) if jaundice is present. 5. Instructing the parents to avoid administering medications unless prescribed. 6. Arranging for indefinite home schooling because the child will not be able to return to school.

341. Answer: 1, 2, 5 Rationale: Hepatitis is an acute or chronic inflammation of the liver that may be caused by a virus, a medication reaction, or another disease process. Because hepatitis can be viral, standard precautions should be instituted in the hospital. The child should be discouraged from sharing toys, so playtime in the playroom with other children is not part of the plan of care. The child will be allowed to return to school 1 week after the onset of jaundice, so indefinite home schooling would not need to be arranged. Jaundice is an expected finding with hepatitis and would not 1004 warrant notification of the PHCP. Provision of a low-fat, well-balanced diet is recommended. Parents are cautioned about administering any medication to the child, because normal doses of many medications may become dangerous owing to the liver's inability to detoxify and excrete them. Hand washing is the most effective measure for control of hepatitis in any setting, and effective hand washing can prevent the immunocompromised child from contracting an opportunistic type of infection.

372. The nurse reviews the record of a child who is suspected to have glomerulonephritis. Which statement by the child's parent should the nurse expect that is associated with this diagnosis? 1. "His pediatrician said his kidneys are working well." 2. "I noticed his urine was the color of cola lately." 3. "I'm so glad they didn't find any protein in his urine." 4. "The nurse who admitted my child said his blood pressure was low."

372. Answer: 2 Rationale: Glomerulonephritis refers to a group of kidney disorders characterized by inflammatory injury in the glomerulus. Gross hematuria, resulting in dark, smoky, cola-colored or brown-colored urine, is a classic symptom of glomerulonephritis. Blood urea nitrogen levels and serum creatinine levels may be elevated, indicating that kidney function is compromised. A mild to moderate elevation in protein in the urine is associated with glomerulonephritis. Hypertension is also common because of fluid volume overload secondary to the kidneys not 1099 working properly.

373. The nurse performing an admission assessment on a 2-year-old child who has been diagnosed with nephrotic syndrome notes that which most common characteristic is associated with this syndrome? 1. Hypertension 2. Generalized edema 3. Increased urinary output 4. Frank, bright red blood in the urine

373. Answer: 2 Rationale: Nephrotic syndrome is defined as massive proteinuria, hypoalbuminemia, hyperlipemia, and edema. Other manifestations include weight gain; periorbital and facial edema that is most prominent in the morning; leg, ankle, labial, or scrotal edema; decreased urine output and urine that is dark and frothy; abdominal swelling; and blood pressure that is normal or slightly decreased.

374. The nurse is planning care for a child with hemolytic-uremic syndrome who has been anuric and will be receiving peritoneal dialysis treatment. The nurse should plan to implement which measure? 1. Restrict fluids as prescribed. 2. Care for the arteriovenous fistula. 3. Encourage foods high in potassium. 4. Administer analgesics as prescribed.

374. Answer: 1 Rationale: Hemolytic-uremic syndrome is thought to be associated with bacterial toxins, chemicals, and viruses that result in acute kidney injury in children. Clinical manifestations of the disease include acquired hemolytic anemia, thrombocytopenia, renal injury, and central nervous system symptoms. A child with hemolytic-uremic syndrome undergoing peritoneal dialysis because of anuria would be on fluid restriction. Pain is not associated with hemolytic-uremic syndrome, and potassium would be restricted, not encouraged, if the child is anuric. Peritoneal dialysis does not require an arteriovenous fistula (only hemodialysis).

375. A 7-year-old child is seen in a clinic, and the pediatrician documents a diagnosis of nighttime (nocturnal) enuresis. The nurse should provide which information to the parents? 1. Nighttime (nocturnal) enuresis does not respond to treatment. 2. Nighttime (nocturnal) enuresis is caused by a psychiatric problem. 3. Nighttime (nocturnal) enuresis requires surgical intervention to improve the problem. 4. Nighttime (nocturnal) enuresis is usually outgrown without therapeutic intervention.

375. Answer: 4 Rationale: Nighttime (nocturnal) enuresis occurs in a child who has never been dry at night for extended periods. The condition is common in children, and most children eventually outgrow bedwetting without therapeutic intervention. The child is unable to sense a full bladder and does not awaken to void. The child may have delayed maturation of the central nervous system. The condition is not caused by a psychiatric problem.

376. The nurse provided discharge instructions to the parents of a 2-year-old child who had an orchiopexy to correct cryptorchidism. Which statement by the parents indicates the need for further instruction? 1. "I'll check his temperature." 2. "I'll give him medication so he'll be comfortable." 3. "I'll check his voiding to be sure there's no problem." 4. "I'll let him decide when to return to his play activities."

376. Answer: 4 Rationale: Cryptorchidism is a condition in which 1 or both testes fail to descend through the inguinal canal into the scrotal sac. Surgical correction may be necessary. All vigorous activities should be restricted for 2 weeks after surgery to promote healing and prevent injury. This prevents dislodging of the suture, which is internal. Normally, 2-year-olds want to be active; allowing the child to decide when to return to his play activities may prevent healing and cause injury. The parents should be taught to monitor the temperature, provide analgesics as needed, and monitor the urine output.

377. The nurse is reviewing a treatment plan with the parents of a newborn with hypospadias. Which statement by the parents indicates their understanding of the plan? 1. "Caution should be used when straddling my infant on a hip." 2. "Vital signs should be taken daily to check for bladder infection." 3. "Catheterization will be necessary when my infant does not void." 4. "Circumcision has been delayed to save tissue for surgical repair."

377. Answer: 4 Rationale: Hypospadias is a congenital defect involving abnormal placement of the urethral orifice of the penis. In hypospadias, the urethral orifice is located below the glans penis along the ventral surface. The infant should not be circumcised, because the dorsal foreskin tissue will be used for surgical repair of the hypospadias. Options 1, 2, and 3 are unrelated to this disorder.

378. The nurse is caring for an infant with a diagnosis of bladder exstrophy. To protect the exposed bladder tissue, the nurse should plan which intervention? 1. Cover the bladder with petroleum jelly gauze. 2. Cover the bladder with a nonadhering plastic wrap. 3. Apply sterile distilled water dressings over the bladder mucosa. 4. Keep the bladder tissue dry by covering it with dry sterile gauze.

378. Answer: 2 Rationale: In bladder exstrophy, the bladder is exposed and external to the body. In this disorder, one must take care to protect the exposed bladder tissue from drying, while allowing the drainage of urine. This is accomplished best by covering the bladder with a nonadhering plastic wrap. The use of petroleum jelly gauze should be avoided, because this type of dressing can dry out, adhere to the mucosa, and damage the delicate tissue when removed. Dry sterile dressings and dressings soaked in solutions (that can dry out) also damage the mucosa when removed.

380. The nurse collects a urine specimen preoperatively from a child with epispadias who is scheduled for surgical repair. When analyzing the results of the urinalysis, which should the nurse most likely expect to note? 1. Hematuria 2. Proteinuria 3. Bacteriuria 4. Glucosuria

380. Answer: 3 Rationale: Epispadias is a congenital defect involving abnormal placement of the urethral orifice of the penis. The urethral opening is located anywhere on the dorsum of the penis. This anatomical characteristic facilitates entry of bacteria into the urine. Options 1, 2, and 4 are not characteristically noted in this condition.

381. The nurse is performing an assessment on a child admitted to the hospital with a probable diagnosis of nephrotic syndrome. Which assessment findings should the nurse expect to observe? Select all that apply. 1. Pallor 2. Edema 3. Anorexia 4. Proteinuria 5. Weight loss 6. Decreased serum lipids

381. Answer: 1, 2, 3, 4 Rationale: Nephrotic syndrome is a kidney disorder characterized by massive proteinuria, hypoalbuminemia, edema, elevated serum lipids, anorexia, and pallor. The child gains weight.

Following diagnosis of Wilms' tumor, the child undergoes removal of the affected kidney. In the postoperative period, priority nursing assessments should focus on: 1. The incision. 2. Lung sounds. 3. Temperature. 4. Kidney function.

9 Answer: 4All of these assessments look at possible postoperative complications. Since the child is left with only one kidney, failure of that kidney due to inadequate blood flow, infection, or any other cause could be fatal.

Which is the priority nursing action when preparing a neonate born with a gastroschisis defect for transport to a pediatric hospital for corrective surgery? 1. Covering the exposed intestines with sterile moist gauze 2. Wrapping the newborn warmly in two or three blankets 3. Providing a sterile water feeding to maintain hydration during transport 4. Allowing the parents of the newborn to see their child prior to transport

Answer: 1 Explanation: 1. It is important to keep the intestine from drying during transport. 2. Placement in a transport isolette would be preferred to wrapping due to the nature of the birth defect. 3. The newborn should be NPO. 4. While it is important for the parents to see their child before transport, this is not the priority nursing intervention.

A newborn diagnosed with an omphalocele defect is admitted to the intensive care nursery. Which nursing action is appropriate based on the current data? 1. Placing the newborn on a radiant warmer 2. Placing the newborn in an open crib 3. Preparing the newborn for phototherapy 4.Preparing the newborn for a bottlefeeding

Answer: 1 Explanation: 1. Omphalocele is a congenital malformation in which intra-abdominal contents herniate through the umbilical cord. The newborn loses heat through the viscera; a warmer is indicated to prevent hypothermia. 2. The crib would not provide adequate maintenance of temperature control. 3. Phototherapy is used to treat hyperbilirubinemia, not an omphalocele defect. 4. The newborn will require surgical correction of the defect prior to initiating bottle or breast feeding.

A child returns from exploratory surgery following a gunshot wound to the abdomen. Which nursing intervention should be excluded for the plan of care? 1. Immediate initiation of oral feedings 2. Assessment of the surgical site 3. Administration of opioid narcotics for pain management 4. Visitation at the bedside

Answer: 1 Explanation: 1. The child will be NPO after an exploratory abdominal surgery. The nurse should exclude this from the child's plan of care. 2. The surgical site must be visualized frequently for bleeding. 3. Pain management is essential and opioid analgesics are often necessary after exploratory surgery. 4. This describes family-centered care; parents should be involved as much as possible and should be present before the child wakes up.

The nurse is unsuccessful in inserting a nasogastric tube for a newborn client. The nurse suspects the newborn has esophageal atresia/tracheoesophageal (EA/TE) fistula. Which nursing action is appropriate while waiting for the healthcare provider to further assess the neonate? 1. Position the newborn in semi-Fowler position. 2. Allow the newborn to be taken to the mother's room for bonding. 3. Offer the newborn formula feeding instead of breastfeeding. 4. Wrap the newborn in blankets and place in a crib by the viewing window.

Answer: 1 Explanation: 1. This will reduce stomach juices from being aspirated into the lungs. 2. Because an anomaly is suspected, the newborn should remain under visualization until the diagnosis is confirmed and medical orders determined. 3. If an EA/TE fistula is suspected, the feeding should be withheld until the diagnosis is confirmed or cleared. 4. A newborn wrapped in blankets cannot be observed clearly. The child should be placed in an over-bed warmer.

A child experienced a lacerated spleen in a motor vehicle accident. Which is the highestpriority nursing intervention on admission to the pediatric intensive care unit (PICU) following surgery? 1. Observing for signs of hypovolemic shock 2. Maintaining IV fluids 3. Implementing strict bedrest 4. Administering blood products as ordered

Answer: 1 Explanation: 1. The priority nursing intervention is observing for signs of hypovolemic shock due to bleeding from the lacerated spleen. The other interventions are appropriate but not the highest priority.

A school-age client diagnosed with nephrotic syndrome is severely edematous. The primary healthcare provider has placed the child on bed rest. Which nursing intervention is a priority for this client? 1. Reposition the child every 2 hours. 2. Monitor BP every 30 minutes. 3. Encourage fluids. 4. Limit visitors.

Answer: 1 Explanation: 1. A child with severe edema, on bed rest, is at risk for altered skin integrity. To prevent skin breakdown, the child should be repositioned every 2 hours. Vital signs are taken every 4 hours, fluids need to be monitored and should not be encouraged, and the child needs social interaction, so visitors should not be limited.

A child, in renal failure, is diagnosed with hyperkalemia. Which food choices will the nurse teach the parents and child to avoid? 1. Carrots and green, leafy vegetables 2. Chips, cold cuts, and canned foods 3. Spaghetti and meat sauce, breadsticks 4. Hamburger on a bun, cherry gelatin

Answer: 1 Explanation: 1. Carrots and green, leafy vegetables are high in potassium. Chips, cold cuts, and canned foods are high in sodium but not necessarily potassium. Spaghetti and meat sauce with breadsticks and a hamburger on a bun with cherry gelatin would be acceptable choices for a lowpotassium diet.

The nurse is caring for a 5-month-old with biliary atresia. The mother asks why the healthcare provider wants her child to take the medication, cholestyramine. What would the nurse's response be? 1. Decrease itching 2. Increase WBCs 3. Decrease use of antibiotics 4. Increase appetite

Answer: 1 Explanation: 1. Cholestyramine is taken to decrease itching.

The nurse is providing care to a male infant who is diagnosed with hypospadias. Which clinical manifestation does the nurse anticipate when assessing this infant? 1. A urethral meatus that is located on the ventral surface of the penis 2. The presence of foreskin 3. A small opening or a fissure that extends the entire length of the penis 4. An opening on the dorsal surface of the penis

Answer: 1 Explanation: 1. For an infant diagnosed with hypospadias, the nurse would anticipate a urethral meatus that is located on the ventral surface of the penis. Infants diagnosed with hypospadias may also have a partial absence of the foreskin. A small opening or a fissure that extends the entire length of the penis or an opening on the dorsal side of the penis would be expected for an infant diagnosed with epispadias.

A child is scheduled for a kidney transplant. The nurse completes the preoperative teaching to prepare the child and parents for the surgery and postoperative considerations. Which statement by the parents indicates understanding of the teaching session? 1. "We know it's important to see that our child takes prescribed medications after the transplant." 2. "We'll be glad we won't have to bring our child in to see the doctor again." 3. "We're happy our child won't have to take any more medicine after the transplant." 4. "We understand our child won't be at risk anymore for catching colds from other children at school."

Answer: 1 Explanation: 1. It is important that the nurse emphasize compliance with treatments that will need to be followed after the transplant. Follow-up appointments will be necessary, as well as medications and general health promotion.

The nurse is administering several medications to an infant with neurologic impairment and delay. Which medication is a proton pump inhibitor that is administered for gastroesophageal reflux? 1. Omeprazole 2. Ranitidine 3. Phenytoin 4. Glycopyrrolate

Answer: 1 Explanation: 1. Omeprazole is the proton pump inhibitor that blocks the action of acidproducing cells and is used to treat gastroesophageal reflux. Ranitidine causes the stomach to produce less acid and may be used to treat gastroesophageal reflux, but it is a histamine-2 receptor blocker. Phenytoin is an anticonvulsant used to treat seizures, and glycopyrrolate is an anticholinergic agent used to inhibit excessive salivation.

An infant born with an omphalocele defect is admitted to the intensive-care nursery. Which instruction from the nurse manager to the unlicensed assistive personnel is most appropriate? 1. Prepare a warmer. 2. Prepare a crib. 3. Prepare a feeding of formula. 4. Prepare the bilirubin light.

Answer: 1 Explanation: 1. Omphalocele is a congenital malformation in which intra-abdominal contents herniate through the umbilical cord. The infant many lose heat through the viscera; a warmer is indicated to prevent hypothermia. The crib would not provide adequate maintenance of temperature control. The infant is NPO (nothing by mouth) preoperatively and may or may not need a bilirubin light before surgery.

A preschool-age client diagnosed with nephrotic syndrome is placed on prednisone for several weeks. Which teaching point is appropriate for the nurse to include in the teaching plan for this client? 1. Never stop the medication suddenly. 2. This drug is taken once a week on Sunday. 3. The child should always take the medication at night before bed. 4. This drug should be taken with meals.

Answer: 1 Explanation: 1. Prednisone, a corticosteroid with anti-inflammatory action, is frequently used to treat nephrotic syndrome. It should never be stopped suddenly. The drug is taken more than once a week and can be taken any time during the day, but should remain on a constant schedule. Taking with food is always appropriate for most medications, but it does not have to be with a meal.

A 3-day-old preterm infant is diagnosed with necrotizing enterocolitis. The nurse plans care around the frequent radiographs. How frequently should the nurse anticipate that the radiology staff will bring the portable machine to the nursery? 1. Every 6 hours 2. Every 12 hours 3. Every 24 hours 4. Every 48 hours

Answer: 1 Explanation: 1. Radiographs are done every 6 hours to evaluate for perforation.

A neonate is fed 20 mL of formula every three hours by orogastric lavage. At the beginning of this feeding, the nurse aspirates 15 mL of gastric residual. Which action by the nurse is the most appropriate? 1. Withhold the feeding and notify the healthcare provider. 2. Replace the residual and continue with the full feeding. 3. Replace the residual but only give 5 mL of the feeding. 4. Withhold the feeding and check the residual in three hours.

Answer: 1 Explanation: 1. Residual of more than half the amount of feeding indicates a feeding intolerance and could be a sign of necrotizing enterocolitis. Early detection of enterocolitis is essential, and aggressive management is required. Therefore, the healthcare provider should be notified of this finding. The amount of residual is too much to replace and continue with the feeding, and waiting for 3 hours to recheck the residual could delay treatment of a serious condition.

An infant is born with an esophageal atresia and tracheoesophageal fistula. Which preoperative nursing diagnosis is the priority for this infant? 1. Risk for Aspiration Related to Regurgitation 2. Acute Pain Related to Esophageal Defect 3. Ineffective Infant Feeding Pattern Related to Uncoordinated Suck and Swallow 4. Ineffective Tissue Perfusion: Gastrointestinal, Related to Decreased Circulation

Answer: 1 Explanation: 1. With the most common type of esophageal atresia and tracheoesophageal fistula, the upper segment of the esophagus ends in a blind pouch and a fistula connects the lower segment to the trachea. Preoperatively, there is a risk of aspiration of gastric secretions from the stomach into the trachea because of the fistula that connects the lower segment of the esophagus to the trachea. Pain is not usually experienced preoperatively with this condition. The infant is always kept NPO (nothing by mouth) preoperatively, so ineffective feeding pattern would not apply. Tissue perfusion is not a problem with this condition.

Which menu choices for a child who is diagnosed with renal failure and experiencing hyperkalemia indicate the need for further instruction by the nurse? 1. Carrots and green, leafy vegetables 2. Spaghetti and meat sauce with breadsticks 3. Hamburger on a bun and cherry gelatin 4. Chips, cold cuts, and canned foods

Answer: 1 Explanation: 1. Carrots and green, leafy vegetables are high in potassium. 2. Spaghetti and meat sauce with breadsticks would be acceptable choices for a lowpotassium diet. 6 Copyright © 2017 Pearson Education, Inc. 3. Hamburger on a bun with cherry gelatin would be acceptable choices for a lowpotassium diet. 4. Chips, cold cuts, and canned foods are high in sodium but not necessarily in potassium.

The nurse is preparing medication instruction for a child who has undergone a kidney transplant and is prescribed cyclosporine. The parents ask the nurse about the reason for the cyclosporine. Which rationale for this medication should the nurse include in the response? 1. Suppress rejection 2. Decrease pain 3. Improve circulation 4. Boost immunity

Answer: 1 Explanation: 1. Cyclosporine is given to suppress rejection. 2. Cyclosporine does not decrease pain. 3. Cyclosporine does not affect circulation. 4. Cyclosporine does not boost immunity.

Which clinical manifestations should the nurse anticipate when assessing a child who has been admitted to the hospital unit with a diagnosis of minimal change nephrotic syndrome (MCNS)? 1. Massive proteinuria, hypoalbuminemia, and edema 2. Hematuria, bacteriuria, and weight gain 3. Urine specific gravity decreased and urinary output increased 4. Gross hematuria, albuminuria, and fever

Answer: 1 Explanation: 1. Nephrotic syndrome is an alteration in kidney function secondary to increased glomerular basement membrane permeability to plasma protein. It is characterized by massive proteinuria, hypoalbuminemia, and edema. 2. Bacteriuria and fever are associated with a urinary tract infection. Because of the edema, a weight gain, not a weight loss, would be seen. 3. In MCNS, the urine output decreases and the specific gravity of urine increases. 4. Gross hematuria and hypertension are associated with glomerulonephritis.

A preschool-age child is admitted to the hospital with acute postinfectious glomerulonephritis (APIGN) and is admitted to the hospital. Which is the priority nursing diagnosis for this child? 1. Risk for Injuryrelated to hypertension. 2. Altered Growth and Developmentrelated to a chronic disease. 3. Risk for Infectionrelated to hypertension. 4. Fluid Volume Excessrelated to decreased plasma filtration.

Answer: 1 Explanation: 1. The child with APIGN has marked hypertension, which can lead to cardiac failure and cerebral injuries. 2. Growth and development are not normally affected because this is an acute process, not a chronic one. 3. While a risk for infection might be present, it is not related to the hypertension. 4. Although fluid retention occurs, this is not the priority diagnosis.

) Which are the leading causes of pediatric abdominal injuries for which the nurse should provide client teaching during scheduled health maintenance visits? Select all that apply. 1. Motor vehicle crashes 2. Falls 3. Blunt trauma 4. Stabbing 5. Impalement

Answer: 1, 2, 3 Explanation: 1. Motor vehicle crashes are a leading cause of pediatric abdominal injuries. The nurse should provide education related to proper use of seat belts during health maintenance visits to decrease the incidence of abdominal injuries. 2. Falls are a leading cause of pediatric abdominal injuries. The nurse should include education regarding age-appropriate pediatric fall prevention during health maintenance visits. 3. Blunt trauma is a leading cause of pediatric abdominal injuries. The nurse should include prevention strategies during health maintenance visits. 4. While stabbing can cause abdominal injury, this is not a common cause in the pediatric population. 5. While impalement can cause abdominal injury, this is not a common cause in the pediatric population.

A child is prescribed hemodialysis for the treatment of kidney failure. When providing care for this child, what will the nurse monitor for during the assessment? Select all that apply. 1. Shock 2. Hypotension 3. Infections 4. Migraines 5. Fluid overload

Answer: 1, 2, 3 Explanation: 1. Rapid changes in fluid and electrolyte balance during hemodialysis may lead to shock and hypotension. Other complications to watch for are thromboses and infection. Migraines and fluid overload are not clinical manifestations associated with hemodialysis.

Which nutritional interventions should the nurse include in the plan of care for a pediatric client who is receiving peritoneal dialysis in the treatment of chronic renal failure? Select all that apply. 1. Provide small, frequent meals. 2. Avoid battles over nutritional intake. 3. Administer supplements by tube feedings, if needed. 4. Implement hand hygiene frequently. 5. Perform daily catheter site care.

Answer: 1, 2, 3 Explanation: 1. The child will feel full with smaller amounts of food because of the dialysate. 2. The child will be more inclined to eat if there is less stress. 3. Adequate nutrition is important for growth and development, and must be supported if oral intake is inadequate. 4. This intervention is appropriate to prevent infection; it is not a nutritional intervention. 14 Copyright © 2017 Pearson Education, Inc. 5. This intervention is appropriate to prevent infection; it is not a nutritional intervention.

7 The nurse has completed discharge teaching on the dietary regimen of a child with celiac disease. The nurse recognizes that client education has been successful when the mother states that the child must comply with the gluten-free diet: 1. Throughout life. 2. Until the child has achieved all major developmental milestones. 3. Only until all symptoms are resolved. 4. Until the child has reached adolescence.

Answer: 1Discharge planning focuses on educating the parents in maintaining a gluten-free diet for the child. Dietary modifications are lifelong and should not be discontinued when the child is symptom-free. Symptoms will return if dietary restrictions are not maintained.

Which actions should the nurse implement when assessing the physical growth for a child who is diagnosed with chronic renal failure? Select all that apply. 1. Asking the child to step on the scale 2. Measuring the child's height 3. Measuring the child's head circumference 4. Using the Denver II with the child 5. Monitoring the child's blood pressure

Answer: 1, 2, 3 Explanation: 1. Weight is a physical growth assessment parameter the nurse uses for a child diagnosed with chronic renal failure. 2. Height is a physical growth assessment parameter the nurse uses for a child diagnosed with chronic renal failure. 3. Head circumference is a physical growth assessment parameter the nurse uses for a child diagnosed with chronic renal failure. 4. The Denver II is a developmental assessment tool. It is not used to assess physical growth. 5. Blood pressure is not a criterion used to measure physical growth.

Which risks of undescended testes should the nurse include in the teaching session for the parents of a newborn diagnosed with this condition? Select all that apply. 1. Sperm production will be affected after puberty. 2. Abdominal testes are subject to injury. 3. Abdominal testes have a higher risk of developing cancer. 4. Hormonal production will be affected. 5. The testes are at greater risk of torsion.

Answer: 1, 2, 3, 5 Explanation: 1. Sperm production by abdominal testes is affected by the heat of the body. 2. Positioning of the testes in the scrotum reduces the risk of injury. 3. Statistics have shown this statement is correct. 4. Production of hormones is not affected by the location of the testes. 5. Abdominal testes have a higher risk of twisting on its blood supply.

Which factors in the maternal medical history should cause the nurse concern regarding the development of cleft lip or cleft palate during pregnancy? Select all that apply. 1. Cigarette smoking 2. Alcohol use 3. Excessive folate intake 4. Glucocorticoid use 5. Anticoagulant use

Answer: 1, 2, 4 Explanation: 1. Cigarette smoking during pregnancy is a risk factor for cleft lip and cleft palate. 2. Alcohol use during pregnancy is a risk factor for cleft lip and cleft palate. 3. Excessive folate intake is not a risk factor for cleft lip and cleft palate. A folate deficiency is often the cause for these disorders. 4. Glucocorticoid use is a risk factor for cleft lip and cleft palate. 5. Anticoagulant use is not a risk factor for cleft lip and cleft palate.

The nurse is preparing an educational session for sexually active adolescents. Which statements are appropriate for the nurse to include when educating about sexually transmitted infections (STIs)? Select all that apply. 1. "Frequently diagnosed STIs include chlamydia, genital herpes, gonorrhea, human papillomavirus, trichomoniasis, and syphilis." 2. "Your risk for contracting an STI can be decreased by using a condom when having sex." 3. "Birth control pills are useful in decreasing your risk of contracting an STI." 4. "Risk factors for pelvic inflammatory disease (PID) include multiple sexual partners, lack of barrier protection during intercourse, and history of an STI." 5. "Pelvic inflammatory disease (PID) is an infection of the lower genital tract."

Answer: 1, 2, 4 Explanation: 1. It is appropriate for the nurse to include the frequently diagnosed STIs, the fact that the risk is decreased by using a condom, and the risk factors for pelvic inflammatory disease. Birth control pills are useful in decreasing the risk of pregnancy but are not useful for decreasing the risk of contracting an STI. PID is an infection of the upper genital tract, not the lower genital tract.

Which statements should the nurse include in a presentation related to the general function of the gastrointestinal (GI) system for parents of pediatric clients? Select all that apply. 1. "The GI tract is responsible for the ingestion and absorption of food." 2. "Newborns have smaller stomachs but increased peristalsis." 3. "All children require smaller, more frequent feedings." 4. "Infants lack certain digestive enzymes which increases the risk for regurgitation." 5. "By the second year of life a child is able to accommodate three meals each day."

Answer: 1, 2, 5 Explanation: 1. This statement is correct. The GI system is responsible for the ingestion and absorption of food. 2. This statement is correct. Newborns have smaller stomachs but an increased rate of peristalsis. 3. This statement is false. All children do not require smaller, more frequent feedings. This statement is true for newborns and infants. 4. This statement is false. While infants do lack certain digestive enzymes, this does not increase regurgitation but causes abdominal distention due to gas. 5. This statement is true. By the second year of life children are able to accommodate three meals each day.

Which assessment questions should the nurse include in the psychosocial assessment to determine the effects of chronic renal failure treatments on the growth and development of a school-age child? Select all that apply. 1. "How does it make you feel to have to follow a special diet?" 2. "Do you take your medications every day?" 3. "How does it make you feel to undergo dialysis treatments?" 4. "Do you attend school each day?" 5. "How does it make you feel when your parents come home late from work?"

Answer: 1, 3 Explanation: 1. School-age children are often embarrassed about being seen as different from peers. It is appropriate for the nurse to ask the child how it feels to have to follow a special diet. 2. While it is important to assess medication use, this question is not appropriate for the psychosocial portion of the assessment. 3. School-age children are often embarrassed about being seen as different from peers. It is appropriate for the nurse to ask the child how it feels to have to undergo dialysis treatments. 4. While it is important to determine if the child attends school every day, this question is not appropriate for the psychosocial portion of the assessment. 5. This question will not help the nurse to determine the effects of the treatments for chronic renal failure on the child's growth and development.

Which topics should the nurse include in discharge instructions related to enhanced safety for a pediatric client who experienced an abdominal injury after a biking accident? Select all that apply. 1. Use of hand signals 2. Age-appropriate use of child safety seats 3. Age-appropriate bicycles 4. Use of a helmet 5. Avoid assigning blame

Answer: 1, 3, 4 Explanation: 1. Information related to appropriate hand signals when riding a bicycle is an injury prevention strategy that the nurse should include in the teaching session. 2. The use of an age-appropriate child safety seat is not an appropriate discharge instruction for a child who experienced an abdominal injury after a biking accident. 3. Information related to an age-appropriate bicycle is an injury prevention strategy that the nurse should include in the teaching session. 4. Information related to the use of a helmet is an injury prevention strategy that the nurse should include in the teaching session. 5. While the nurse should avoid assigning blame when providing care for a child who experienced an abdominal injury as a result of a biking accident, this is not an appropriate injury prevention topic to include in the discharge teaching session.

The nurse educator is preparing an in-service on the basic functions of the gastrointestinal (GI) system. Which statements will the nurse educator include in the in-service? Select all that apply. 1. "The GI system is responsible for the ingestion of fluids and nutrients." 2. "The GI system is responsible for the excretion of fluids and nutrients." 3. "The GI system is responsible for the metabolism of nutrients." 4. "As infants grow, their stomach capacity increases, decreasing the frequency with which they need to be fed." 5. "By the second year of life, digestive processes are still developing."

Answer: 1, 3, 4 Explanation: 1. The GI system is responsible for the ingestion of fluids and nutrients as well as the metabolism of nutrients. As infants grow, their stomach capacity increases, which does decrease the frequency with which they need to be fed. The GI system is responsible for the excretion of waste products. By the second year of life, digestive processes are fairly complete.

A neonate is born with a bilateral cleft lip that was not detected during the pregnancy. The parents are distressed about the appearance of their infant. Which nursing actions are appropriate to assist the parents to bond with their newborn? Select all that apply. 1. Calling the newborn by the chosen name 2. Keeping the newborn's lower face covered with the blanket 3. Smiling and talking to the newborn in the parents' presence 4. Showing the parents before and after pictures of other children with cleft lips 5. Discussing positive features of the baby

Answer: 1, 3, 4, 5 Explanation: 1. This behavior humanizes the child to the parents and is appropriate. 2. This indicates that the infant's appearance is distressing. Although the nurse would want to shield the child from a visitor's stare, the nurse would not want to hide the child from her own family. 3. This indicates acceptance of the infant by the nurse. 4. It is usually not appropriate to show before and after pictures as you cannot predict the success of the surgery on this child. But in the case of cleft lip, the improvement will be significant and it is considered acceptable to show before and after pictures. 5. Statements like, "Your baby is the sweetest thing—she never cries," can help the parents recognize positive features about their baby.

The nurse is caring for a school-age client who had an appendectomy after a ruptured appendix. Which orders does the nurse anticipate for this client? Select all that apply. 1. Antibiotics 2. A clear liquid diet 3. NG tube 4. Vital signs every 4 hours 5. Frequent monitoring of bowel sounds

Answer: 1, 3, 4, 5 Explanation: 1. Antibiotics, an NG tube, vital signs every 4 hours, and frequent monitoring of bowel sounds are appropriate interventions following a ruptured appendix. The client is NPO until bowel sounds return.

he nurse educator is teaching a group of nursing students the pathophysiologic reasons related to genitourinary (GU) disorders in the pediatric population. Which statements are appropriate for the nurse educator to include in the teaching session? Select all that apply. 1. "Incomplete organ development during fetal development is the cause of many GU disorders." 2. "Improper placement of the urethra in vagina is one cause of GU disorders." 3. "GU disorders in the pediatric population can be caused by hydronephrosis." 4. "GU disorders in the pediatric population are not caused by infections." 5. "Anatomic obstruction or incomplete nerve innervation can cause GU disorders."

Answer: 1, 3, 5 Explanation: 1. Pathophysiologic causes of GU disorders in the pediatric population include incomplete organ development during fetal development; hydronephrosis; and anatomic obstruction or incomplete nerve innervations. Improper placement of the urethra in the penis, not the vagina, is another pathophysiologic cause of GU disorders. GU disorders can also be caused by infection.

Which nursing actions are appropriate to assess growth and development for an adolescent client diagnosed with chronic renal failure? Select all that apply. 1. Using the Denver II during a health maintenance visit 13 Copyright © 2017 Pearson Education, Inc. 2. Educating parents on normal milestones 3. Monitoring for delayed sexual maturation 4. Comparing blood pressure values from previous visit 5. Plotting height and weight measurements

Answer: 1, 3, 5 Explanation: 1. The Denver II is a developmental assessment tool that is appropriate for the nurse to use when assessing growth and development for an adolescent client diagnosed with chronic renal failure. 2. It is appropriate for the nurse to educate the client's parents on normal milestones; however, this is not a nursing assessment. 3. Monitoring for delayed sexual maturation is appropriate when assessing growth and development for an adolescent client diagnosed with chronic renal failure. 4. Blood pressure is not a growth and development parameter. 5. Plotting height and weight measurements is an appropriate nursing action to assess growth and development for an adolescent client diagnosed with chronic renal failure.

18) The child is diagnosed with an upper urinary tract infection (UTI). The family asks the nurse what is the difference in the symptoms of an upper versus a lower urinary tract infection? Match each symptom (1-8 below) with the appropriate infection. A. Lower UTI B. Upper UTI 1. High fever 2. Diarrhea 3. Chills 4. Hematuria 5. Costovertebral angle tenderness 6. Cloudy urine 7. Suprapubic or flank pain 8. Moderate/severe dehydration

Answer: 1/B, 2/A, 3/B, 4/A, 5/B, 6/A, 7/A, 8/B 1. Upper UTI 2. Lower UTI 3. Upper UTI 4. Lower UTI 5. Upper UTI 6. Lower UTI 7. Lower UTI 8. Upper UTI

9 The doctor orders a clean-catch urine specimen on an infant who is not toilet-trained. The best means of collecting this urine would be to: 1. Perform a straight catheterization. 2. Apply a urine collection bag. 3. Use diaper analysis. 4. Perform Foley catheterization.

Answer: 1Clean-catch urine specimens are not reliable urine samples; therefore, catheterization is necessary. The urine does need to be obtained at the time of voiding.

6 While performing a newborn assessment, the nurse notes the infant is having difficulty breathing, with nasal flaring, cyanosis, retractions, and an absence of breath sounds on the left side. The nurse auscultates the apical pulse on the right side of the chest. The nurse notifies the physician immediately because of suspected: 1. Diaphragmatic hernia. 2. Pyloric stenosis. 3. Cleft palate. 4. Omphalocele.

Answer: 1Clinical findings will vary in infants born with congenital diaphragmatic hernias, but the first indications are of respiratory distress. Further assessment will Chapter 13Gastrointestinal Health Problems 335 ANSWERS & RATIONALES reveal bowel sounds auscultated over the chest, cardiac sounds on the right of the chest, and a sunken abdomen with a barrel-shaped chest

3 While a child is receiving prednisone (Deltasone) for treatment of nephrotic syndrome, the nurse determines that it is important to assess the child for: 1. Infection. 2. Urinary retention. 3. Easy bruising. 4. Hypoglycemia.

Answer: 1Prednisone is a synthetic corticosteroid that depresses the immune response and increases susceptibility to infection. Steroids mask infection; therefore, the child must be assessed for subtle signs and symptoms of illness.

8 A child has been admitted to the unit with acute glomerulonephritis. The test that would confirm this diagnosis is: 1. Antistreptolysin-O (ASO) titer. 2. Urinalysis. 3. Blood cultures. 4. White blood cell (WBC) count.

Answer: 1The ASO titer indicates a preceding infection with group-A beta-hemolytic streptococcus.The urinalysis would show hematuria, but this alone would not be diagnostic of acute glomerulonephritis. Blood cultures may be negative as the infection preceded the illness by 1 to 3 weeks

An adolescent client reports recurrent abdominal pain with diarrhea and bloody stools. Which type of inflammatory bowel disease does the nurse suspect based on these data? 1. Necrotizing enterocolitis (NEC) 2. Ulcerative colitis (UC) 3. Crohn disease 4. Appendicitis

Answer: 2 Explanation: 1. NEC is usually seen in premature infants and generally not in an adolescent client. 2. Diarrhea and bloody stools are typical symptoms of UC. 3. The teen with Crohn disease might have abdominal pain and diarrhea, but stools usually do not have blood in them. 4. Appendicitis is not associated with bloody stools and usually not with diarrhea.

A nasogastric tube to suction is ordered for a neonate diagnosed with a diaphragmatic hernia. Which complication related to gastric drainage is the priority when planning care for this neonate? 1. Weight loss 2. Metabolic alkalosis 3. Dehydration 4. Hyperbilirubinemia

Answer: 2 Explanation: 1. Weight loss and inadequate nutrition are not the priority for this client. 2. When large quantities of gastric juice is removed, acid is lost and metabolic alkalosis follows. 3. The volume would not be sufficient to cause dehydration. 4. Hyperbilirubinemia is unrelated to gastric suction.

Which assessment finding would necessitate action by the nurse for a 10-month-old child who is 4 hours postoperative for the placement of a urethral stent? 1. Bloody urine 2. One void since returning from surgery 3. Bladder spasms responding to pharmacologic intervention 4. Double diapering from the previous shift

Answer: 2 Explanation: 1. Bloody urine is expected in the immediate postoperative period. 2. A 10-month-old child will void more often than 1 time in 4 hours. This could indicate the stent is occluded. The surgeon should be notified. 3. This is a normal finding. 4. This is a desired finding and does not need to be reported to the surgeon.

Which assessment finding, after the dialysate is drained during peritoneal dialysis for a child experiencing acute renal failure, would warrant further action by the nurse? 1. The dialysate is clear on return. 2. The volume of drained dialysate is less than the volume infused. 3. The child is restless, wanting to get up and play. 4. The child's vital signs are basically the same as were noted on infusion.

Answer: 2 Explanation: 1. This is a normal finding and does not require reporting. 2. This indicates fluids are being retained and is not desirable. The healthcare provider should be notified. 3. This could indicate the child is feeling better. It is a desired effect and does not require reporting to the healthcare provider. 4. This is an expected finding. No dramatic differences in vital signs should be noted.

A child diagnosed with acute glomerulonephritis is in the playroom and experiences blurred vision and headache. Which action by the nurse is the most appropriate? 1. Check the urine to see if hematuria has increased. 2. Obtain a blood pressure on the child; notify the healthcare provider. 3. Reassure the child, and encourage bed rest until the headache improves. 4. Obtain serum electrolytes, and send a urinalysis to the lab.

Answer: 2 Explanation: 1. Blurred vision and headache may be signs of encephalopathy, a complication of acute glomerulonephritis. A blood pressure (BP) should be obtained and the healthcare provider notified. The healthcare provider may decide to order an antihypertensive to bring down the BP. This is a serious complication, and delay in treatment could mean lethargy and seizures. Therefore, the other options (checking urine for hematuria, encouraging bed rest, and obtaining serum electrolytes) do not directly address the potential problem of encephalopathy.

A child recently had a kidney transplant and is prescribed cyclosporine. The parents ask the nurse about the reason for the cyclosporine. Which reason will the nurse include in the response for why this medication is prescribed? 1. To boost immunity 2. To suppress rejection 3. To decrease pain 4. To improve circulation

Answer: 2 Explanation: 1. Cyclosporine is given to suppress rejection. It doesn't boost immunity, decrease pain, or improve circulation.

A newborn is diagnosed with Hirschsprung disease. Which clinical manifestations found on assessment support this newborn's diagnosis? 1. Acute diarrhea; dehydration 2. Failure to pass meconium; abdominal distension 3. Currant jelly; gelatinous stools; pain 4. Projectile vomiting; altered electrolytes

Answer: 2 Explanation: 1. Hirschsprung disease is the absence of autonomic parasympathetic ganglion cells in the colon that prevent peristalsis at that portion of the intestine. In newborns, the symptoms include failure to pass meconium and abdominal distension. Acute diarrhea and dehydration are symptoms characteristic of gastroenteritis. Currant jelly, gelatinous stools, and pain are symptoms of intussusception, and projectile vomiting and altered electrolytes are symptoms of pyloric stenosis.

The nurse teaches parents that the anticholinergic drug oxybutynin is used to treat enuresis. The parents ask the nurse why the medication is being prescribed. Which response by the nurse is the most appropriate? 1. "It's an antidepressant that is used to help the child relax." 2. "It will help decrease the spasms sometimes associated with enuresis." 3. "It has an antidiuretic effect, so your child can attend sleepovers." 4. "It will slow the production of urine, so your child does not have to urinate as frequently."

Answer: 2 Explanation: 1. Oxybutynin (Ditropan) is an anticholinergic that relaxes the smooth muscle of the bladder and decreases spasms. Oxybutynin is not an antidepressant or an antidiuretic, and does not slow urine production.

A nurse is preparing to admit a child with possible obstructive uropathy. Which laboratory test should the nurse expect to draw on this child? 1. Platelet count 2. Blood urea nitrogen (BUN) and creatinine 3. Partial thromboplastin time (PTT) 4. Blood culture Answer: 2

Answer: 2 Explanation: 1. The blood urea nitrogen (BUN) and creatinine are serum lab tests for kidney function. Obstructive uropathy is a structural or functional abnormality of the urinary system that interferes with urine flow and results in urine backflow into the kidneys; therefore, the BUN and creatinine will be elevated. Platelet count and partial thromboplastin time (PTT) are drawn when a bleeding disorder is suspected. A blood culture is done when an infectious process is suspected.

A preschool-age client is diagnosed with acute glomerulonephritis and is admitted to the hospital. Which nursing diagnosis is most appropriate for this client? 1. Risk for Injury Related to Loss of Blood in Urine 2. Fluid-Volume Excess Related to Decreased Plasma Filtration 3. Risk for Infection Related to Hypertension 4. Altered Growth and Development Related to a Chronic Disease

Answer: 2 Explanation: 1. The fluid is excessive, and fluid and electrolyte balance should be monitored. There is no risk for injury because the blood loss in the urine is not such that it causes anemia. While a risk for infection may be present, it is not related to the hypertension. Growth and development is not normally affected because this is an acute process, not a chronic one.

Which urinalysis result should the nurse anticipate for a child who is admitted with acute glomerulonephritis? 1. Bacteriuria and increased specific gravity 2. Hematuria and proteinuria 3. Proteinuria and decreased specific gravity 4. Bacteriuria and hematuria

Answer: 2 Explanation: 1. Glomerulonephritis is an inflammation of the glomeruli of the kidneys. Bacteriuria is not present. But because the urine is concentrated, the specific gravity is increased. 2. Glomerulonephritis is an inflammation of the glomeruli of the kidneys. The clinical manifestation of glomerulonephritis is grossly bloody hematuria with mild to moderate proteinuria, and because the urine is concentrated, the specific gravity is increased. 3. Glomerulonephritis is an inflammation of the glomeruli of the kidneys. Because the urine is concentrated, the specific gravity is increased. 4. Glomerulonephritis is an inflammation of the glomeruli of the kidneys. Bacteriuria is not present.

Which is the appropriate nursing intervention when providing care to a child, diagnosed with nephrotic syndrome, who is edematous and on bed rest? 1. Monitor blood pressure every 30 minutes. 2. Reposition every 2 hours. 3. Limit visitors. 4. Encourage fluids.

Answer: 2 Explanation: 1. Vital signs are taken every 4 hours. 2 Copyright © 2017 Pearson Education, Inc. 2. A child with severe edema, on bed rest, is at risk for altered skin integrity. To prevent skin breakdown, the child should be repositioned every 2 hours. 3. The child needs social interaction, so visitors should not be limited. 4. Fluids need to be monitored; they should not be encouraged.

Answer: 2 Explanation: 1. Weight loss and inadequate nutrition are not the priority for this client. 2. When large quantities of gastric juice is removed, acid is lost and metabolic alkalosis follows. 3. The volume would not be sufficient to cause dehydration. 4. Hyperbilirubinemia is unrelated to gastric suction.

Answer: 2, 3, 4 Explanation: 1. Fiber should be decreased, not increased, as diarrhea is one of the symptoms of Crohn disease. 2. This is correct information. 3. This is individualizing the diet and is appropriate. 4. This addition provides an easy way to meet the nutritional needs. 5. Stress should be avoided at mealtimes.

The nurse is caring for the newborn with hyperbilirubinemia. What nursing diagnoses would the nurse address? Select all that apply. 1. Activity intolerance 2. Deficient fluid volume 3. Risk for impaired attachment 4. Ineffective breathing pattern 5. Risk for imbalanced body temperature

Answer: 2, 3, 5 Explanation: 1. Nursing diagnoses that may apply to the newborn with hyperbilirubinemia are deficient fluid volume, risk for impaired attachment, risk for imbalanced body temperature, and risk for injury.

Which complications should the nurse monitor for when providing care to a child who is having hemodialysis for the treatment of kidney failure? Select all that apply. 1. Migraines 2. Hypotension 3. Infections 4. Fluid overload 5. Shock

Answer: 2, 3, 5 Explanation: 1. Migraines are not a clinical manifestation associated with hemodialysis. 2. Rapid changes in fluid and electrolyte balance during hemodialysis can lead to hypotension. 3. Infection is another complication that may occur during hemodialysis. 4. Fluid overload is not a clinical manifestation associated with hemodialysis. 5. Rapid changes in fluid and electrolyte balance during hemodialysis can lead to shock.

Which instructions should be provided to the parents of a 4-year-old girl who has experienced chronic urinary tract infections (UTIs) in the last 2 years? Select all that apply. 1. Wear only nylon underwear for better air flow. 2. Teach the child to wipe from front to back. 3. Encourage the child to take long baths by allowing the child bubbles and toys in the tub. 4. Encourage the child to drink additional fluids throughout the day. 5. Plan potty breaks every 2 hours throughout the day.

Answer: 2, 4, 5 Explanation: 1. The child should wear cotton underwear. 2. This prevents bacteria from the rectum from being introduced into the urethra. 3. Bubble baths should be avoided. 4. Extra fluids will "wash" bacteria out of the bladder. 5. Children get so involved in playing that they often hold their urine. Voiding every 2 hours will reduce the time for bacteria to grow in the bladder.

The nurse is providing care to a newborn client who presents in the pediatric clinic for a 2-week health maintenance visit. The parents of the newborn are concerned, as their baby has "gas all the time." Which responses from the nurse are appropriate? Select all that apply. 1. "Your baby has a relaxed lower esophageal sphincter, which is causing the gas." 2. "Your baby lacks the enzyme amylase, which is causing the gas." 3. "Your baby lacks the enzyme insulin, which is causing the gas." 4. "Your baby has an immature liver, which is causing the gas." 5. "Your baby lacks an enzyme that helps to digest fats, which is causing the gas."

Answer: 2, 5 Explanation: 1. Newborns and infants do have a relaxed lower esophageal sphincter; however, this is not responsible for gas but for frequent regurgitation of small amounts of oral feedings. 2. Newborns and infants lack several enzymes that assist with the digestive process. One of these enzymes is amylase, which assists with carbohydrate digestion. The lack of this enzyme causes abdominal distention due to gas. 3. Insulin is not an enzyme and is not lacking in the newborn. 4. While newborns and infants do have immature livers, that is not what is causing the gas. 5. Lipase is a digestive enzyme that assists in fat digestion. Infants and newborns do lack this enzyme, which would cause abdominal distention due to gas.

6 A 14-year-old boy is brought into the Emergency Department with a diagnosis of rule out appendicitis. He is complaining of right lower quadrant pain. The nurse's most appropriate action to assist in managing his pain would be to: 1. Insert a rectal tube. 2. Apply an ice bag. 3. Apply a heating pad. 4. Administer an intravenous antispasmodic agent.

Answer: 2An ice bag may help relieve his pain. A rectal tube is contraindicated because it stimulates bowel motility, which would increase the pain. A heating pad is contraindicated because it increases the flow of blood to the appendix and may lead to rupture. An antispasmodic agent would not be beneficial for the pain associated with appendicitis. Antispasmodic agents are typically used to inhibit smooth muscle contractions.

2 A child has been admitted to the hospital with a diagnosis of "rule out nephrosis." The nurse would assess the child for: 1. Hematuria. 2. Edema. 3. Petechial rash. 4. Dehydration.

Answer: 2Edema is the major clinical symptom of nephrosis. The child may gain twice his or her normal weight in severe cases

6 A child has been diagnosed with acute renal failure secondary to an infectious organism. The nurse would question the medical order for: 1. Aqueous penicillin. 2. Gentamicin (Garamycin). 3. Antihypertensives. 4. Corticosteroids.

Answer: 2Gentamicin is an aminoglycoside antibiotic that is nephrotoxic. Nephrotoxic drugs should be avoided in a child with acute renal failure. The other options do not represent drug groups that are particularly nephrotoxic.

9 The nurse is developing a teaching plan for the parents of an infant diagnosed with hepatitis A. Which of the following instructions would be included to reduce the risk for transmission of this disease? 1. Disinfect all clothing and eating utensils on a daily basis. 2. Tell family members to wash their hands frequently. 3. Spray the yard to eliminate infected insects. 4. Vacuum the carpets and upholstery to rid the house of the infectious host.

Answer: 2Hepatitis A is highly contagious and is transmitted primarily through the fecal-oral route. The virus is transmitted by direct person-to-person contact or through ingestion of contaminated food or water, especially shellfish growing in contaminated water. The remaining answers are related to other infectious diseases.

2 The nurse is caring for an infant vomiting secondary to pyloric stenosis. The mother questions why the vomitus of this child appears different from that of her other children when they have the flu. The nurse would explain that the emesis of an infant with pyloric stenosis does not contain bile because: 1. The GI system is still immature in newborns and infants. 2. The obstruction is above the bile duct. 3. The emesis is from passive regurgitation. 4. The bile duct is obstructed.

Answer: 2In pyloric stenosis, bile is unable to enter the stomach from the duodenum because the pylorus muscle is hypertrophied, which causes the obstruction.

8 An infant returns from initial surgery for Hirschsprung's disease. All of the following are routine postoperative nursing interventions. Because of the type of surgery this child had, the nurse would exclude which of them? 1. Maintaining the child NPO until bowel sounds return 2. Monitoring rectal temperature every 4 hours 3. Reuniting the parents with the child as soon as possible 4. Assessing the surgical site every 2 hours

Answer: 2The corrective surgery for Hirschsprung's disease requires pulling the end of the normal bowel through the muscular sleeve of the rectum. With this type of procedure, rectal temperatures and any invasive procedure would be avoided to allow proper healing to occur

Which assessment data would cause the nurse to suspect that a 3-year-old child has Hirschsprung disease? 1. Clay-colored stools and dark urine 2. History of early passage of meconium in the newborn period 3. History of chronic, progressive constipation and failure to gain weight 4. Continual bouts of foul-smelling diarrhea

Answer: 3 Explanation: 1. Clay-colored stools and dark urine are not associated with Hirschsprung disease. 2. The infant with Hirschsprung disease often has delayed meconium stools. 3. These are symptoms of Hirschsprung disease in an older infant or child. 4. Diarrhea is not typical; obstruction is more likely.

Which parental statement at the end of a teaching session by the nurse indicates correct understanding of colostomy stoma care for the infant client? 1. "We will change the colostomy bag with each wet diaper." 2. "We will expect a moderate amount of bleeding after cleansing the area around the stoma." 3. "We will watch for skin irritation around the stoma." 4. "We will use adhesive enhancers when we change the bag."

Answer: 3 Explanation: 1. Physical or chemical skin irritation can occur if the appliance is changed too frequently, or with each wet diaper. 2. Bleeding is usually attributable to excessive cleaning. 3. Skin irritation around the stoma should be assessed; it could indicate leakage. 4. Adhesive enhancers should be avoided on the skin of infants. Their skin layers are thin, and removal of the appliance can strip off the skin.

Which parental action, observed during a home care visit for an infant diagnosed with gastroesophageal reflux, requires intervention by the nurse? 1. The infant's formula has rice cereal added. 2. The mother holds the infant in a high Fowler position while feeding. 3. After feeding, the infant is placed in a car seat. 4. The mother draws up the ranitidine (Zantac) in a syringe for oral administration.

Answer: 3 Explanation: 1. Rice cereal thickens the formula and helps prevent regurgitation. This is appropriate. 2. This position will help prevent regurgitation and is appropriate. 3. Infant seats are not recommended, as they put pressure on the abdomen and may contribute to regurgitation. 4. Since dosing is small, it is appropriate to use a syringe for accurate measurement.

A toddler is admitted to the surgical unit for a planned closure of a temporary colostomy. Which medical prescription should the nurse question? 1. Clear liquids today. NPO tomorrow 2. Type and cross-match for 1 unit of packed red blood cells. 3. Rectal temperatures every 4 hours 4. Start an intravenous line with D5NS at 20 mL per hour.

Answer: 3 Explanation: 1. This is appropriate in anticipation of surgery. 2. Although not always required during surgery, this would not be inappropriate planning for the surgical procedure. 3. Rectal temperatures are avoided due to the fragile state of the rectum. 4.An IV is appropriate for surgical access

7 The newborn has been diagnosed with cryptorchidism. The physician has ordered human chorionic gonadotropin (HCG) to be administered to the baby. The mother asks the nurse why the baby is receiving this drug. The nurse's best explanation would be that the drug will: 1. Maintain an adequate temperature around the testes. 2. Prevent infections in the undescended testes. 3. Prevent the development of cancer. 4. Promote descent of the testes.

Answer: 4HCG is given to induce the descent of testes if testes have not descended during the first year of life. The other reasons listed are incorrect rationales.

Which is the priority nursing intervention when caring for a neonate who is born with bladder exstrophy? 1. Measuring intake and output 2. Inserting a Foley catheter 3. Covering the defect with sterile plastic wrap 4. Palpating the bladder mass to ensure urine is expelled

Answer: 3 Explanation: 1. Because the bladder constantly drains onto the skin of the abdomen, measuring output is not possible. 2. The bladder is open to the abdomen. A Foley catheter cannot be inserted. 3. This reduces the contamination of the bladder, which should be sterile. 4. The bladder is very sensitive and palpation would cause unnecessary pain.

A child with severe gastroenteritis is admitted to a semiprivate room on the pediatric unit. The charge nurse should place this client with which roommate? 1. An infant with meningitis 2. A child with fever and neutropenia 3. Another child with gastroenteritis 4. A child recovering from an appendectomy

Answer: 3 Explanation: 1. Gastroenteritis may be viral or bacterial and can be infectious. It is best to cohort children with this infectious process. Good handwashing is essential to prevent the spread. An infant with meningitis, a child with fever and neutropenia, and a child recovering from an appendectomy should not be placed with another child with an infectious process.

The nurse is planning postoperative care for an infant after a cleft-lip repair. Which nursing intervention is most appropriate for this infant? 1. Prone positioning 2. Suctioning with a Yankauer device 3. Supine or side-lying positioning 4. Avoidance of soft elbow restraints

Answer: 3 Explanation: 1. Integrity of the suture line is essential for postoperative care of cleft-lip repair. The infant should be placed in a supine or side-lying position to avoid rubbing the suture line on the bedding. The prone position should be avoided. A Yankauer suction device is made of hard plastic and, if used, could cause trauma to the suture line. Suctioning should be done with a small, soft suction catheter. Soft elbow restraints may be used to prevent the infant from touching the incisional area.

A child is admitted to the hospital unit with a diagnosis of minimal-change nephrotic syndrome (MCNS). Which clinical manifestations does the nurse anticipate when conducting the admission assessment? 1. Hematuria, bacteriuria, weight gain 2. Gross hematuria, albuminuria, fever 3. Massive proteinuria, hypoalbuminemia, edema 4. Hypertension, weight loss, proteinuria

Answer: 3 Explanation: 1. Nephrotic syndrome is an alteration in kidney function secondary to increased glomerular basement membrane permeability to plasma protein. It is characterized by massive proteinuria, hypoalbuminemia, and edema. While hematuria and hypertension may be present, they are not pronounced. Gross hematuria and hypertension are associated with glomerulonephritis. Bacteriuria and fever are associated with a urinary tract infection. Because of the edema, a weight gain, not a weight loss, would be seen.

Which clinical manifestations should the nurse anticipate upon assessment for a preschoolage child with a urinary tract infection (UTI)? 1. Headache, hematuria, and vertigo 2. Foul-smelling urine, elevated blood pressure (BP), and hematuria 3. Urgency, dysuria, and fever 4. Severe flank pain, nausea, and headache

Answer: 3 Explanation: 1. Hematuria might be present, but there will be no complaints of headache or vertigo. 2. While foul-smelling urine and hematuria can be present, there is no elevated BP, headache, or vertigo. 3. Clinical manifestations of UTI in a preschool-age child include fever, urgency, and dysuria. 4. There could be flank pain, although the preschooler might be unable to describe it. There will be no complaints of headache.

Which interventions should the nurse include in the plan of care for a pediatric client who is receiving peritoneal dialysis in the treatment of chronic renal failure to prevent infection? Select all that apply. 1. Provide small, frequent meals. 2. Avoid battles over nutritional intake. 3. Administer supplements by tube feedings, if needed. 4. Implement hand hygiene frequently. 5. Perform daily catheter site care.

Answer: 3, 4 Explanation: 1. This intervention is appropriate to meet the child's nutritional needs; however, this will not prevent infection. 2. This intervention is appropriate to meet the child's nutritional needs; however, this will not prevent infection. 3. This intervention is appropriate to meet the child's nutritional needs; however, this will not prevent infection. 4. Aseptic technique reduces chance of introducing bacteria into the abdomen. 5. Skin around the catheter site will have fewer organisms that could potentially cause infection.

Which gastrointestinal defects, often diagnosed shortly after birth, should the nurse include in the assessment process of all newborns? Select all that apply. 1. Pyloric stenosis 2. Biliary atresia 3. Hirschsprung disease 4. Umbilical hernia 5. Diaphragmatic hernia

Answer: 3, 5 Explanation: 1. Pyloric stenosis is not diagnosed in the newborn nursery, but in the 2- to 4-week-old infant. 2. Symptoms of biliary atresia would not be observable until several weeks of age. 3. Symptoms of Hirschsprung disease may be observable in the newborn nursery. 4. Umbilical hernia cannot be diagnosed at birth. 5. Diaphragmatic hernia will show symptoms immediately after birth due to compression of the lung.

3 The nurse is caring for a toddler who is not toilettrained. The doctor has ordered intake and output measurement. The nurse will most accurately measure the urine by: 1. Estimating output as small, moderate, or large and recording on the child's chart. 2. Weighing each wet diaper and recording the weight of the diaper as the amount of urine output. 3. Subtracting the weight of a dry diaper from a wet diaper and record this amount. 4. Determining urine output by the number of diaper changes in each 24-hour period.

Answer: 3Diapers are weighed on a gram scale before using them and after removal (1 g = 1 mL). The weight of the dry diaper is then subtracted from the weight of the wet diaper to determine urine output

1 An 18-month-old child with a history of cleft lip and palate has been admitted for palate surgery. The nurse would provide which explanation about why a toothbrush should not be used immediately after surgery? 1. The toothbrush would be frightening to the child. 2. The child no longer has deciduous teeth. 3. The suture line could be interrupted. 4. The child will be NPO.

Answer: 3During the immediate postoperative period, protecting the operative site is a priority in the nursing care of this child. A toothbrush should be a familiar object to an 18-month-old child. Deciduous (primary) teeth are still present at this age and are replaced by permanent (secondary) teeth around 6 years of age. Oral care will be performed according to the physicians' orders but usually consists of cleansing the area with sterile water

1 A child who underwent cleft palate repair has just returned from surgery with elbow restraints in place. The parents question why their child must have the restraints. The nurse would give which of the following as the best explanation to the parents? 1. "This device is frequently used postoperatively to protect the IV site in small children." 2. "The restraints will help us maintain proper body alignment." 3. "Elbow restraints are used postoperatively to keep children's hands away from the surgical site." 4. "The restraints help maintain the child's NPO status."

Answer: 3Elbow restraints are used to keep hands away from the mouth after cleft palate surgery. This precaution will be maintained at home until the palate is healed, usually 4 to 6 weeks. They are not used to protect the IV site, maintain NPO status, or maintain body alignment

10 Which of the following signs would the nurse recognize as an indication of moderate dehydration in a preschooler? 1. Sunken fontanel 2. Diaphoresis 3. Dry mucous membranes 4. Decreased urine specific gravity

Answer: 3Mucous membranes typically appear dry when moderate dehydration is observed. Other typical findings associated with moderate dehydration include restlessness with periods of irritability (especially infants and young children), rapid pulse, poor skin turgor, delayed capillary refill, and decreased urine output. Both anterior and posterior fontanels are closed on a preschool-age child. The skin is usually dry with decreased elasticity, not diaphoretic. Urine specific gravity increases with decreased urine output associated with dehydration

The nurse instructs the parents about postoperative feeding following their infant's pyloromyotomy. The nurse evaluates that the parents' understand the in structions when the parents state they will: 1. Avoid bubbling the baby after feeding to prevent vomiting. 2. Rock the baby to sleep after feeding to keep the infant calm. 3. Slowly increase the volume offered according to the physician's orders. 4. Maintain the infant on antiemetics to prevent vomiting.

Answer: 3The goal after pyloromyotomy is to slowly increase the volume of feeding while preventing vomiting. Bubbling is essential after feed. Rocking is avoided as this might increase vomiting. Antiemetics are not helpful as the vomiting is not associated with nausea.

1 The nurse would place highest priority on which nursing activity in managing a young child diagnosed with urinary tract infection (UTI)? 1. Provide adequate nutrition to prevent dehydration. 2. Prevent enuresis. 3. Administer ordered antibiotics on schedule. 4. Restrict fluids to provide kidney rest.

Answer: 3Urinary tract infections are ascending in nature; an untreated UTI can lead to acute pyelonephritis with resulting kidney scarring and damage. Early diagnosis and prompt antimicrobial therapy will prevent or minimize permanent renal damage.

5 The parents of a child diagnosed with upper urinary tract infection (UTI) ask the nurse why the child needs a daily weight. In formulating a response, the nurse includes that it is important because a daily weight will: 1. Determine if the child's caloric intake is adequate. 2. Indicate the need for dietary restrictions of sodium and potassium. 3. Keep track of possible loss or gain of fluid retained in body tissues. 4. Track the amount of fluid ingested orally each day.

Answer: 3With infectious or inflammatory processes of the upper urinary tract, the kidneys' ability to filter and reabsorb salt and water is altered, resulting in edema. Weights can be an easy and effective measure to determine fluid loads

0 A 14-year-old is being treated for renal failure. The nurse would ensure that the child follows a: 1. High-sodium diet. 2. High-protein diet. 3. Low-sodium diet. 4. Low-fiber diet.

Answer: 3With the inability to secrete urine, electrolytes will build up in the blood, including sodium and potassium. The child should be on a low-sodium, lowpotassium diet with restricted fluids and proteins

A nurse is preparing for the delivery of a newborn with a known diaphragmatic hernia defect. Which equipment should the nurse have on hand for the delivery? 1. Bag-valve-mask system 2. Sterile gauze and saline 3. Soft arm restraints 4. Endotracheal tube

Answer: 4 Explanation: 1. A bag-valve-mask system, or Ambu bag, could push air into the stomach and cause abdominal distension, increase pressure on the diaphragm, and impair breathing. 2. The defect is not external, so sterile gauze and saline are not needed. 3. Soft arm restraints might be necessary but at are not an immediate concern. 4. A diaphragmatic hernia (protrusion of abdominal contents into the chest cavity through a defect in the diaphragm) is a life-threatening condition. Intubation is required immediately so that the newborn's respiratory status can be stabilized.

The nurse is planning care for a school-age client who is postoperative for the surgical removal of the appendix. In addition to pharmacologic pain management, which should the nurse include in the plan of care to address pain? 1. Applying a warm, moist pack every 4 hours 2. Applying EMLA cream to the incision site prior to ambulation 3. Applying a cold, moist pack every 2 hours 4. Applying a pillow against the abdomen to splint the incision site when coughing

Answer: 4 Explanation: 1. Heat and moisture are not used on the incision area, as they can impair the healing process of the wound. 2. EMLA cream is a medication that requires a prescription. 3. Heat and ice are not used on the incision area, as they can impair the healing process of the wound. 4. A splint pillow placed on the abdomen is a nonpharmacologic strategy to decrease discomfort after an appendectomy.

The nurse is providing care for a pediatric client who is diagnosed with a Wilms tumor. Which laboratory test result should the nurse monitor prior to administering the prescribed chemotherapy dose? 1. Hemoglobin 2. Red blood cell count 3. Platelets 4. Absolute neutrophil count (ANC)

Answer: 4 Explanation: 1. Hemoglobin indicates oxygen-carrying capacity, not immune response. 2. Red blood cell count has no correlation with immune function. 3. Platelets are associated with clotting, not immune function. 4. The absolute neutrophil count uses both the segmented (mature) and bands (immature) neutrophils as a measure of the body's infection-fighting capability.

4 A high school experiences an outbreak of hepatitis B. In teaching the high school students about hepatitis B, the school nurse would explain: 1. Hepatitis B cannot exist in a carrier state. 2. Hepatitis B is primarily transmitted through the fecaloral route. 3. Immunity to all types of hepatitis will occur after this current attack. 4. Hepatitis B can be prevented by receiving the HBV vaccine.

Answer: 4HBV vaccine provides active immunity, and current recommendations include immunizations for all newborns, as well as several high-risk groups. Hepatitis B is spread by blood and body fluids, including sexual contact, not the fecal-oral route. The disease can exist in a carrier state.

The nurse is providing care to a pediatric client, diagnosed with inflammatory bowel disease, who is prescribed daily prednisone. Which parental statement regarding administration of this drug indicates correct understanding of the teaching provided by the nurse? 1. "I will administer this medication between meals." 2. "I will administer this medication at bedtime." 3. "I will administer this medication one hour before meals." 4. "I will administer this medication with meals."

Answer: 4 Explanation: 1. Prednisone can cause gastric irritation and should not be given on an empty stomach. 2. Prednisone can cause gastric irritation and should not be given before bedtime on an empty stomach. 3. Prednisone can cause gastric irritation and should not be given on an empty stomach one hour before meals. 4. Prednisone, a corticosteroid, can cause gastric irritation. It should be administered with meals to reduce the gastric irritation.

The nurse is assessing abdominal girth for a pediatric client who presents with abdominal distension. Which nursing action is appropriate? 1. Measuring the girth just below the umbilicus 2. Measuring the girth just below the sternum 3. Measuring the girth just above the pubic bone 4. Measuring the girth around the portion of the stomach

Answer: 4 Explanation: 1. The circumference below the umbilicus would not be an accurate abdominal girth. 2. The circumference just below the sternum would not be an accurate abdominal girth. 3. The circumference just above the pubic bone would not be an accurate abdominal girth. 4. An abdominal girth should be taken around the largest circumference of the abdomen, just above the umbilicus.

Which is the priority nursing diagnosis when planning care for a newborn who is born with esophageal atresia and tracheoesophageal fistula? 1. Ineffective Tissue Perfusion 2. Ineffective Infant Feeding Pattern 3. Acute Pain 4. Risk for Aspiration

Answer: 4 Explanation: 1. Tissue perfusion is not a primary problem with this condition. 2. The infant is always kept NPO (nothing by mouth) preoperatively, so ineffective feeding pattern would not apply. 3. Pain is not usually experienced preoperatively with this condition. 4.This is the most common type of esophageal atresia and tracheoesophageal fistula, where the upper segment of the esophagus ends in a blind pouch and a fistula connects the lower segment to the trachea. Preoperatively, there is a risk of aspiration of gastric secretions from the stomach into the trachea because of the fistula that connects the lower segment of the esophagus to the trachea.

Which clean-catch urinalysis finding should the nurse be most concerned for a child who is admitted to an urgent care center to rule out a urinary tract infection? 1. 2+ white blood cells 2. 1+ red blood cells 3. Urine appearance: cloudy 4. Specific gravity: 1009

Answer: 4 Explanation: 1. White blood cells are expected. 2. Red blood cells are common in the urine of a child with a urinary tract infection. 3. With white blood cells in the urine, this is a common finding. 4. This is a very dilute urine. With white blood cells (WBCs), red blood cells (RBCs), and bacteria in the urine, you would expect the urine to contain more solutes.

The nurse is evaluating an infant's tolerance of feedings after a pyloromyotomy. Which finding indicates that the infant is not tolerating the feeding? 1. Need for frequent burping 2. Irritability during feeding 3. The passing of gas 4. Emesis after two feedings

Answer: 4 Explanation: 1. An infant is not tolerating feedings after a pyloromyotomy if emesis is present. Frequent burping, irritability, and the passing of gas would be expected findings following a pyloromyotomy and would indicate tolerance of the feeding.

Which symptoms are characteristic of a preschool-age client who is diagnosed with a urinary tract infection? 1. Foul-smelling urine, elevated blood pressure, and hematuria 2. Severe flank pain, nausea, headache 3. Headache, hematuria, vertigo 4. Urgency, dysuria, fever

Answer: 4 Explanation: 1. Clinical manifestations of a urinary tract infection (UTI) in a preschool-age client include fever, urgency, and dysuria. While hematuria may be present, there is no elevated blood pressure, headache, or vertigo.

A child with inflammatory bowel disease is prescribed prednisone daily. At which time is it most appropriate for the family to administer the prednisone? 1. Between meals 2. One hour before meals 3. At bedtime 4. With meals

Answer: 4 Explanation: 1. Prednisone, a corticosteroid, can cause gastric irritation. It should be administered with meals to reduce the gastric irritation.

A school-age client is recovering after abdominal surgery. The nurse is planning care for the return of bowel function. Which intervention should be included in the client's plan of care? 1. Fowler's position 3 times per day for 30 minutes each time 2. Assist the child in choosing a low-fat diet. 3. Commode at bedside 4. Ambulate 3 to 4 times a day.

Answer: 4 Explanation: 1. The best data that indicate return of bowel sounds are flatus and passage of stool. Ambulation is the primary intervention to assist with both. A Fowler's position, bedside commode, and a low-fat diet will not assist with bowel function.

Which parental statement indicates understanding of the process involved with a kidney transplant for a child with renal failure? 1. "We are happy our child will not have to take any more medicine after the transplant." 2. "We understand our child will not be at risk anymore for catching colds from other children at school." 3. "We will be glad we will not have to bring our child in to see the doctor again." 4. "We know it is important to see that our child takes prescribed medications after the transplant."

Answer: 4 Explanation: 1. Medications and general health promotion will be necessary. 2. The child will be on immunosuppressing drugs and will be at increased risk for colds and other illnesses. 3. Follow-up appointments will be necessary, as well as medications and general health promotion. 4. It is important that the nurse emphasizes compliance with treatments that will need to be followed after the transplant.

7 The nurse has taught dietary restrictions to a 7-yearold child who has celiac disease. After teaching, the child is allowed to choose a meal from the hospital menu. The nurse evaluates that teaching was effective when the child chooses which of the following? 1. Beef and barley soup, rice cakes, and celery 2. Ham and cheese sandwich with lettuce and tomato on rye toast 3. Beef patty on a hamburger bun and home fries 4. Baked chicken, green beans, and a slice of cornbread

Answer: 4Celiac disease is characterized by intolerance for gluten. Gluten is found in wheat, barley, rye, and oats. This includes bread, cake, doughnuts, cookies, and crackers, as well as processed foods that contain gluten as filler.

8 An appropriate nursing assessment of an infant suspected of having necrotizing enterocolitis would be: 1. pH of the stomach contents. 2. Neurological status every 2 hours. 3. Rectal temperature every 2 hours. 4. Abdominal girth every 4 hours.

Answer: 4Measuring the abdominal girth frequently aids in early detection of necrotizing enterocolitis, which, in turn, minimizes loss of bowel. Assessment of gastric pH is not done. Frequent assessment of the neurologic status is not specific to this disease. Rectal temperatures are contraindicated because of the increased risk of perforation.

3 Immediately after the delivery of an infant with an omphalocele, the nurse would take which of the following actions? 1. Weigh the infant. 2. Insert an orogastric tube. 3. Call the blood bank for 2 units of blood. 4. Cover the sac with moistened sterile gauze.

Answer: 4Omphaloceles are congenital malformations in which abdominal contents protrude through the umbilical cord. The protrusion is covered by a translucent sac; immediately after birth, the sac requires priority attention. The sac is covered with sterile gauze soaked in normal saline solution to prevent drying and injury.

5 A 6-week-old infant is brought into the pediatrician's office with a history of frequent vomiting after feedings and failure to gain weight. The diagnosis of gastroesophageal reflux is made and discharge instructions are begun. While planning discharge teaching on feeding techniques with the parents, the nurse should include instructions to: 1. Dilute the formula. 2. Delay burping to prevent vomiting. 3. Change from milk-based formula to soy-based formula. 4. Position the infant at a 30- to 45-degree angle after feedings.

Answer: 4Small, frequent feedings followed by placing the infant at a 30- to 45-degree angle has been shown to be beneficial in treating gastroesophageal reflux. Diluting the formula would not be recommended because the infant needs the calories from the full-strength formula. It may be recommended to thicken the formula with rice cereal. It is recommended to burp frequently; to delay burping would only increase the occurrences of reflux. Gastroesophageal reflux is not related to milk intolerance so changing the formula would not help the child.

4 The nurse is admitting a 12-year-old girl to the hospital prior to surgery. The physician has ordered a urinalysis. In order to obtain accurate urinalysis data, the nurse should: 1. Encourage fluids to 1000 mL prior to specimen collection. 2. Cleanse the specimen container with povidone-iodine (Betadine) prior to collecting the specimen. 3. Allow the urine to cool to room temperature before taking it to the lab. 4. Provide client/parent education for specimen collection before the specimen is obtained.

Answer: 4Specimens collected utilizing proper technique will minimize contamination of the urine sample ensuring accurate urinalysis results. It is unnecessary to force fluids prior to specimen collection. The specimen container is not cleansed, although the urinary meatus is. The specimen should be sent to the lab immediately after collection to prevent urine degradation.

7 An appropriate nursing diagnosis for a toddler with unrepaired exstrophy of the bladder would be: 1. Disorganized infant behavior. 2. Sexual dysfunction. 3. Urinary retention. 4. Risk for infection.

Answer: 4The open bladder allows bacteria to enter the urinary system, and urinary tract infections are common. At this age, sexual dysfunction would not be an appropriate diagnosis. The unformed bladder does not hold urine, so urinary retention would not be an appropriate diagnosis. Disorganized behavior doesn't apply

9 A child has recurrent nephrotic syndrome. The mother reports to the nurse that she is overwhelmed with the care of her child. After the nurse discusses options with the mother, which statement by the mother indicates continued coping difficulties? 1. "I joined a support group like you suggested. I hope it does some good." 2. "I'm going to ask my mother-in-law to come on a regular basis to allow me an afternoon out." 3. "My husband has agreed to help me manage my son's medication." 4. "We're going to skip his dietary restrictions one day a week to allow us both some relaxation."

Answer: 4The parents must understand the need for compliance with medical orders to promote the child's health. Relaxation should be accomplished without harming the child.

6 A urinalysis is ordered for a child with a throat culture positive for group-A beta-hemolytic streptococcus(strep throat). The mother asks why this test is being ordered. The nurse explains: 1. The urinalysis will indicate whether an HIV infection is also present. 2. Urinary tract infections are common with streptococcal infections and need to receive prompt treatment. 3. Pyelonephritis is a potential complication of antibiotic therapy. 4. Group-A beta-hemolytic streptococcusinfections can be followed by the complication of acute glomerulonephritis.

Answer: 4Urinalysis allows for early diagnosis and treatment of acute glomerulonephritis, which is a serious complication that can follow group-A beta-hemolytic streptococcal infection.

11. The nurse takes care of a newborn diagnosed with Eagle-Barrett syndrome. Which comment by the parent indicates teaching was effective? a. "As a teen, my child might develop end-stage renal disease." b. "My infant has about 3 months to live due to severe renal problems." c. "The skin of his bottom looks like a prune due to poor peristalsis." d. "He has this syndrome from a recessive gene; my next baby will have it too."

Answer: a. "As a teen, my child might develop end-stage renal disease." Feedback: Children with Eagle-Barrett syndrome (prune belly) will develop end-stage renal disease in childhood or adolescence because of inadequate renal function. The skin covering the abdominal wall is thin and resembles a wrinkled prune. Death occurs in the neonatal period due to pulmonary hypoplasia and severe renal dysfunction. Prune belly syndrome is thought to be related to a fetal urinary tract obstruction or a specific injury, not genetics.

12. The mother of a 6-year-old calls the clinic because her child is wetting the bed. Which assessment question by the nurse is most important? a. "Is there a family history of renal or urinary problems?" b. "What happens when the child wets the bed?" c. "At what age was the child potty-trained?" d. "How is the child doing in school?"

Answer: a. "Is there a family history of renal or urinary problems?" Feedback: Enuresis more often occurs in children who have a positive family history, so the primary assessment question is to determine whether there is a family history. The other questions are important to ask when assessing a child with enuresis, but are not the priority.

8. Assessment of a 2-year-old by a nurse in the emergency department reveals the following: edema, hematuria, hypertension, and oliguria. What would the nurse assess as the most likely cause of these symptoms? a. Acute renal failure b. Urinary tract infection c. Vesicoureteral reflux d. Bladder exstrophy

Answer: a. Acute renal failure Feedback: There are several things that can cause acute renal failure, including hemolytic uremic syndrome, nephritic syndrome, and severe dehydration. Most children with acute renal failure are admitted to a pediatric intensive care unit. A urinary tract infection would not cause any of the listed symptoms. Bladder exstrophy is a congenital defect discovered at birth. Vesicoureteral reflux is a backflow of urine from the bladder to the kidneys.

1. Which instructions would the nurse provide to the family of a child who has undergone a hypospadias repair? (Select all that apply.) a. Avoid tub baths until the catheter is removed. b. Notify the primary healthcare provider if there is blood in the urine. c. The child should avoid the straddling position with play. d. It is important that the catheter be left in place. e. Notify the primary healthcare provider if the child goes more than 30 minutes without urine output.

Answer: a. Avoid tub baths until the catheter is removed; c. The child should avoid the straddling position with play; d. It is important that the catheter be left in place. Feedback: The nurse should discuss with the family the importance of leaving the catheter in place, notifying the primary healthcare provider if the child goes more than 1 hour without urine output, notifying the primary healthcare provider if the child avoids the straddling position with play, and avoiding baths until the catheter is removed. It is normal to see blood-tinged urine for several days after surgery.

9. A community health nurse is educating a high school class about sexually transmitted infections (STIs). Which information should be included in the presentation? a. Chlamydia can be asymptomatic. b. Ejaculation must occur for gonorrhea to be transmitted. c. A condom will protect teenagers from getting herpes. d. Intercourse is the only means of transmitting STIs.

Answer: a. Chlamydia can be asymptomatic. Feedback: Abstinence from all forms of sexual contact will protect a teenager from getting an STI. Many people have Chlamydia without knowing it, as it can be asymptomatic. A condom does not always protect a teenager from getting herpes, because the herpes lesion might not be covered by the condom and the condom might break. Gonorrhea can be transmitted without ejaculation.

6. Which intervention would be appropriate when a nurse is caring for a child with acute postinfectious glomerulonephritis (APIGN)? a. Screen family members for strep throat. b. Offer a high-protein diet. c. Maintain strict fluid restriction. d. Monitor the child for hyperactivity.

Answer: a. Screen family members for strep throat. Feedback: Rationale: The child with APIGN should have a diet low in protein with no added salt. Family members should be checked for strep throat, and the child should be monitored for any neurological changes.

2. Which statement by the parent of an uncircumcised male infant would indicate the need for further teaching? a. "Frequent diaper changes are important." b. "I should forcibly retract the foreskin once a day." c. "Once the foreskin is retractable, it should be returned to its normal position after cleaning." d. "Harsh soaps should be avoided."

Answer: b. "I should forcibly retract the foreskin once a day." Feedback: The parent should never force the foreskin to retract, due to the fact that it may cause paraphimosis, which is where the foreskin cannot be returned to its normal position. Harsh soaps should be avoided. Frequent diaper changes are important to prevent irritation, and once the foreskin is retractable in early childhood, always return it to its normal position after cleaning.

4. Which issue is important to discuss when educating a family about nocturnal enuresis? a. Limit daytime fluids. b. Have the child double-void before going to bed. c. Administer laxatives daily. d. Refer the child to counseling immediately.

Answer: b. Have the child double-void before going to bed. Feedback: Counseling is not always indicated. Promoting regular stools and having the child double-void before bed are appropriate interventions. Limiting daytime fluids has not been shown to be effective.

5. A child with nephrotic syndrome is placed on corticosteroids. About which side effects of corticosteroids should the nurse educate the family? a. Impaired balance b. Moon face c. Decreased appetite d. Hair loss

Answer: b. Moon face Feedback: Side effects of corticosteroids include moon face, increased hair growth, increased appetite, and mood swings. Impaired balance is not associated with corticosteroids.

3. Which condition in males would the nurse assess as a medical emergency? a. Cryptorchidism b. Testicular torsion c. Phimosis d. Inguinal hernia

Answer: b. Testicular torsion Feedback: Testicular torsion is a medical emergency and should be surgically repaired within 4-6 hours of onset. The testis rotates on its spermatic cord, obstructing blood supply. Inguinal hernia is when a portion of abdominal cavity protrudes into the groin. It is usually repaired after 3 months of age and is not considered emergent unless the hernia is incarcerated. Cryptorchidism is when a testicle is not descended. It is present at birth, and if the testicle does not descend by 1-2 years of age, it is repaired. Phimosis is when the skin around the glans of the penis is not retractable by young childhood.

10. A nurse is evaluating a parent performing a clean intermittent catheterization on a pediatric client. What would be an indication that the parent needs additional teaching? a. The parent uses a size 4 catheter for the procedure. b. The parent states that the child should be awakened once during the night to be catheterized. c. The parent uses a water-soluble lubricant to coat the end of the catheter. d. The parent uses gentle pressure to advance the catheter if resistance is met.

Answer: b. The parent states that the child should be awakened once during the night to be catheterized. Feedback: It is necessary to perform intermittent catheterization every 3-4 hours but not while the child is sleeping at night. A size 4 or 5 catheter is used for the procedure. A water-soluble lubricant, not Vaseline, is used. In males, the sphincter muscle located at the entrance to the bladder will cause resistance to the catheter, but with gentle pressure, the catheter will advance into the bladder.

7. A nurse is assessing a 3-year-old for hemolytic uremic syndrome (HUS). Which assessment finding would be most characteristic of HUS? a. Fever b. Severe cough c. Diarrhea d. Oliguria

Answer: d. Oliguria Feedback: HUS is characterized by the classic triad of symptoms: thrombocytopenia, hemolytic anemia, and acute renal failure. Severe cough, fever, or diarrhea alone is not a sign of HUS. The problem usually is preceded by a urinary tract infection, upper respiratory infection, or acute gastroenteritis 1-2 weeks prior to the HUS.

4 The nurse is teaching the parents of a preschooler information about urinary tract infections (UTIs) and means of reducing their recurrence. Statements from the parents that indicate an understanding of ways to prevent UTIs include: (Select all that apply.) 1. "I should try to get her to drink a lot of water and juices." 2. "I will buy her underwear made with cotton." 3. "Soaking in a bubble bath will reduce meatal irritation." 4. "If I notice her starting to wet the bed again, I need to have her checked for another urinary tract infection." 5. "I should avoid giving her cranberry juice as it has been shown to make the urine more acidic."

Answers: 1, 2, 4Bubble baths are irritating to the meatus and increase the incidence of urinary tract infections. An acidic urine is desirable in preventing urinary tract infections, and the proper way to wipe is front to back

5 A 4-month-old infant is admitted to the nursing unit with moderate dehydration. Which of the following symptoms does the nurse suspect led to the diagnosis of moderate dehydration in this child? (Select all that apply.) 1. Elevated heart rate 2. Urine specific gravity of 1.038 3. Weight gain 4. Polyuria 5. Slow capillary refill

Answers: 1, 2, 5The nurse would expect an increased desire to drink fluids and a higher specific gravity caused by the concentration of urine. The heart rate would be elevated, and the fontanels sunken. The degree of dehydration is based on the percent of weight loss, so a weight gain would not be likely. Diminished urine output with elevated specific gravity is an expected normal finding in dehydration. Capillary refill is slowed, especially in children under 2 years of age.

4 While gathering admission data on a 16-month-old child, the nurse notes all the following abnormal findings. Which finding is related to a diagnosis of Hirschsprung's disease? (Select all that apply.) 1. Bile-stained vomitus 2. Decreased urine output 3. Poor weight gain since birth 4. Intermittent sharp pain 5. Alternating constipation and diarrhea

Answers: 1, 3, 5Infants with Hirschsprung's disease usually display failure to thrive, poor weight gain, and delayed growth. Vomiting is usually bile stained. The child will demonstrate alternating constipation and diarrhea, but the stools are not bloody. Decreased urine output and intermittent sharp pain are nonspecific symptoms that can be associated with many different diseases and disorders

5 The nurse would include which of the following in the care of a child with acute glomerulonephritis? (Select all that apply.) 1. Careful handling of edematous extremities 2. Observing the child for evidence of hypertension 3. Providing fun activities for the child on bedrest 4. Monitoring the urine for hematuria 5. Encouraging fluid intake

Answers: 2, 3, 4Although children with acute glomerulonephritis may feel well, they are confined to bed until hematuria resolves. This can lead to boredom, making it important for the nurse to provide activities that are fun for the child to help pass the time.

The nurse is teaching the parents of a child with celiac disease about the dietary restrictions. The nurse would explain that the most appropriate diet for their child is a diet that is free of which of the following? (Select all that apply.) 1. Rice 2. Wheat 3. Oats 4. Barley 5. Corn

Answers: 2, 3, 4Most children who remain on a glutenfree diet remain healthy and free of symptoms and complications. Gluten is a protein found in wheat, barley, rye, and oats. For this reason, appropriate foods need to be free of these grains.

1 A newborn is found to have exstrophy of the bladder. The nurse should evaluate the infant for: (Select all that apply.) 1. Hypospadias. 2. Epispadias. 3. Cryptorchidism. 4. Acute tubular necrosis. 5. Bilateral inguinal hernias.

Answers: 2, 5Epispadias and bilateral inguinal hernias are frequent anomalies associated with exstrophy of the bladder. The other conditions listed are not

8 The nurse admits children with the following diseases to the unit. The nurse determines that the children with which diseases are at risk for the development of acute renal failure (ARF)? (Select all that apply.) 1. Leukemia 2. Cryptorchidism 3. Nephrotic syndrome 4. Phenylketonuria 5. Urinary tract infection

Answers: 3, 5Nephrotic syndrome is an inflammatory reaction in the kidneys. Urinary tract infections and obstructions are also associated with the development of acute renal failure. The other diseases pose minimal risk of developing acute renal failure.

A nurse is preparing for the delivery of a newborn with a known diaphragmatic hernia defect. Which equipment does the nurse ensure is prepared at the bedside? 1. Intubation setup 2. Appropriate bag and mask 3. Sterile gauze and saline 4. Soft arm restraints

Explanation: 1. A diaphragmatic hernia (protrusion of abdominal contents into the chest cavity through a defect in the diaphragm) is a life-threatening condition. Intubation is required immediately so the newborn's respiratory status can be stabilized. A bag and mask will not be adequate to ventilate a newborn with this condition. The defect is not external, so sterile gauze and saline are not needed. Soft arm restraints are not immediately necessary.

2 When reviewing a urinalysis report of a client with acute glomerulonephritis, the nurse expects to note which of the following? 1. Decreased creatinine clearance 2. Decreased specific gravity 3. Proteinuria 4. Decreased erythrocyte sedimentation rate (ESR)

nswer: 3Proteinuria (presence of protein in urine) is a prime manifestation of acute glomerulonephritis. The other options are inconsistent with this diagnosis.

The nurse is providing instruction to the parents of an infant with a colostomy. Which statement by the parents indicates appropriate understanding of the teaching session? 1. "We will change the colostomy bag with each wet diaper." 2. "We will use adhesive enhancers when we change the bag." 3. "We will watch for skin irritation around the stoma." 4. "We will expect a moderate amount of bleeding after cleansing the area around the stoma." Answer: 3

xplanation: 1. Skin irritation around the stoma should be assessed; it may indicate leakage. Physical or chemical skin irritation may occur if the appliance is changed too frequently or with each wet diaper. Adhesive enhancers should be avoided on the skin of newborns. Their skin layers are thin, and removal of the appliance can strip off the skin. Also, adhesive contains latex, and its constant use is not advised due to risk of latex allergy development. Bleeding is usually attributable to excessive cleaning.


Set pelajaran terkait

stress management exam questions

View Set

Chapter 69 (Unit 6 Interventions II)

View Set

Business Information Systems Final

View Set

computing final exam study guide

View Set